Menu Close

Question-28456




Question Number 28456 by mondodotto@gmail.com last updated on 26/Jan/18
Commented by Tinkutara last updated on 26/Jan/18
5
Answered by Rasheed.Sindhi last updated on 26/Jan/18
f(2)=5,f(3)=4,g(2)=5,g(3)=2,g(4)=1  .−.−.−.−.  f( g(3) )=f(2)    ∵ g(3)=2                  =5          ∵ f(2)=5
$$\mathrm{f}\left(\mathrm{2}\right)=\mathrm{5},\mathrm{f}\left(\mathrm{3}\right)=\mathrm{4},\mathrm{g}\left(\mathrm{2}\right)=\mathrm{5},\mathrm{g}\left(\mathrm{3}\right)=\mathrm{2},\mathrm{g}\left(\mathrm{4}\right)=\mathrm{1} \\ $$$$.−.−.−.−. \\ $$$$\mathrm{f}\left(\:\mathrm{g}\left(\mathrm{3}\right)\:\right)=\mathrm{f}\left(\mathrm{2}\right)\:\:\:\:\because\:\mathrm{g}\left(\mathrm{3}\right)=\mathrm{2} \\ $$$$\:\:\:\:\:\:\:\:\:\:\:\:\:\:\:\:=\mathrm{5}\:\:\:\:\:\:\:\:\:\:\because\:\mathrm{f}\left(\mathrm{2}\right)=\mathrm{5} \\ $$

Leave a Reply

Your email address will not be published. Required fields are marked *